Sr Examen

Otras calculadoras:

Límite de la función n/log(n)^3

cuando
v

Para puntos concretos:

Gráfico:

interior superior

Definida a trozos:

Solución

Ha introducido [src]
     /   n   \
 lim |-------|
n->oo|   3   |
     \log (n)/
$$\lim_{n \to \infty}\left(\frac{n}{\log{\left(n \right)}^{3}}\right)$$
Limit(n/log(n)^3, n, oo, dir='-')
Método de l'Hopital
Tenemos la indeterminación de tipo
oo/oo,

tal que el límite para el numerador es
$$\lim_{n \to \infty} n = \infty$$
y el límite para el denominador es
$$\lim_{n \to \infty} \log{\left(n \right)}^{3} = \infty$$
Vamos a probar las derivadas del numerador y denominador hasta eliminar la indeterminación.
$$\lim_{n \to \infty}\left(\frac{n}{\log{\left(n \right)}^{3}}\right)$$
=
$$\lim_{n \to \infty}\left(\frac{\frac{d}{d n} n}{\frac{d}{d n} \log{\left(n \right)}^{3}}\right)$$
=
$$\lim_{n \to \infty}\left(\frac{n}{3 \log{\left(n \right)}^{2}}\right)$$
=
$$\lim_{n \to \infty}\left(\frac{\frac{d}{d n} \frac{n}{3}}{\frac{d}{d n} \log{\left(n \right)}^{2}}\right)$$
=
$$\lim_{n \to \infty}\left(\frac{n}{6 \log{\left(n \right)}}\right)$$
=
$$\lim_{n \to \infty}\left(\frac{\frac{d}{d n} \frac{n}{6}}{\frac{d}{d n} \log{\left(n \right)}}\right)$$
=
$$\lim_{n \to \infty}\left(\frac{n}{6}\right)$$
=
$$\lim_{n \to \infty}\left(\frac{n}{6}\right)$$
=
$$\infty$$
Como puedes ver, hemos aplicado el método de l'Hopital (utilizando la derivada del numerador y denominador) 3 vez (veces)
Gráfica
Respuesta rápida [src]
oo
$$\infty$$
Otros límites con n→0, -oo, +oo, 1
$$\lim_{n \to \infty}\left(\frac{n}{\log{\left(n \right)}^{3}}\right) = \infty$$
$$\lim_{n \to 0^-}\left(\frac{n}{\log{\left(n \right)}^{3}}\right) = 0$$
Más detalles con n→0 a la izquierda
$$\lim_{n \to 0^+}\left(\frac{n}{\log{\left(n \right)}^{3}}\right) = 0$$
Más detalles con n→0 a la derecha
$$\lim_{n \to 1^-}\left(\frac{n}{\log{\left(n \right)}^{3}}\right) = -\infty$$
Más detalles con n→1 a la izquierda
$$\lim_{n \to 1^+}\left(\frac{n}{\log{\left(n \right)}^{3}}\right) = \infty$$
Más detalles con n→1 a la derecha
$$\lim_{n \to -\infty}\left(\frac{n}{\log{\left(n \right)}^{3}}\right) = -\infty$$
Más detalles con n→-oo